410 exam 2

अब Quizwiz के साथ अपने होमवर्क और परीक्षाओं को एस करें!

A client's potassium level is elevated. The nurse is reviewing the ECG tracing. Identify the area on the tracing where the nurse would expect to see peaks. Click to select the correct part of the image.

Potassium influences cardiac muscle activity. Alterations in potassium levels change myocardial irritability and rhythm. Hyperkalemia is very dangerous; cardiac arrest can occur. Cardiac effects of elevated serum potassium are significant when the level is above 8 mEq/L. Hyperkalemia causes skeletal muscle weakness and even paralysis, related to a depolarization block in t muscle. Therefore, ventricular conduction is slowed. The earliest change that can be seen are peaked, narrow T waves on the ECG.

A volume-depleted patient would present with which of the following diagnostic lab results? a. BUN-to-creatinine ratio of 24:1 b. Urinary output of 1.2 L/24 hours c. Urine specific gravity of 1.02 d. Capillary refill time of 3 seconds

a A BUN-to-serum creatinine concentration ratio greater than 20:1 is indicative of volume depletion. The other results are within normal range.

An older adult client is prescribed a diuretic for heart failure. What should the nurse include when teaching this client about ways to avoid hypokalemia? a. avoid foods and beverages with caffeine b. limit the intake of green leafy vegetables c. plan to take the medication before bedtime d. take the medication with coffee every morning

a A high caffeine intake can heighten the hypokalemic effects of diuretics. Green leafy vegetables will not help prevent hypokalemia. Taking the medication before bedtime will not impact the development of hypokalemia. Coffee contains caffeine and would potentiate the development of hypokalemia.

The nurse is administering an insulin drip to a patient in ketoacidosis. What insulin does the nurse know can be used intravenously? a. short-acting b. rapid acting c. intermediate-acting d. long-acting

a Insulins may be grouped into several categories based on the onset, peak, and duration of action. Short-acting, also known as regular insulin, is the only insulin administered by IV.

A patient has a serum osmolality of 250 mOsm/kg. The nurse knows to assess further for: a. Dehydration. b. Hyperglycemia. c. Hyponatremia. d. Acidosis.

c Decreased serum sodium is a factor associated with decreased serum osmolality. Dehydration and hyperglycemia are associated with increased serum osmolality; acidosis is associated with increased urine osmolality.

A nurse correctly identifies a urine specimen with a pH of 4.3 as being which type of solution? a. Neutral b. Alkaline c. Acidic d. Basic

c Normal urine pH is 4.5 to 8.0; a value of 4.3 reveals acidic urine pH. A pH above 7.0 is considered an alkaline or basic solution. A pH of 7.0 is considered neutral.

An older adult client is prescribed a beta blocker. Before administration, the nurse will question the prescription if the client has which healh problem? a. osteoarthritis b. lymphedema c. benign prostatic hypertrophy d. chronic obstructive pulmonary disease

d A client with chronic obstructive pulmonary disease is at high risk for adverse effects if prescribed a beta blocker. A beta blocker does not cause adverse effects in the client with osteoarthritis, lymphedema, or benign prostatic hypertrophy.

What clinical indication of hyperphosphatemia does the nurse assess in a patient? a. Bone pain b. Paresthesia c. Seizures d. Tetany

d Tetany is a symptom of hyperphosphatemia. Bone pain, paresthesia, and seizures are associated with hypophosphatemia.

An adult child caring for an older adult parent calls the nurse, stating concern about the parent's chronic bad breath and stubborn plaque on the teeth. Which teeth-cleaning measure should the nurse recommend for this older adult? a. "Use a soft swab to clean the teeth as well as gums." b. "Lemon-glycerin swabs should be used to clean around the gumline." c. "An alcohol-based mouthwash will kill bacteria causing the odor." d. "An manual toothbrush is most effective when giving oral hygiene."

d The use of a toothbrush is more effective than swabs or other soft devices in improving gingival tissues and removing soft debris from the teeth. Lemon-glycerin swabs dry the oral mucosa and contribute to tooth enamel erosion. Mouthwashes with high alcohol content can be too harsh for the mouths of older adults.

What percentage of potassium excreted daily leaves the body by way of the kidneys? a. 20 b. 40 c. 60 d. 80

d To maintain the potassium balance, the renal system must function, because 80% of the potassium excreted daily leaves the body by way of the kidneys. The other numerical values are incorrect.

When the nurse is caring for a patient with type 1 diabetes, what clinical manifestation would be a priority to closely monitor? a. Hypoglycemia b. Hyponatremia c. Ketonuria d. Polyphagia

a The therapeutic goal for diabetes management is to achieve normal blood glucose levels (euglycemia) without hypoglycemia while maintaining a high quality of life.

Hypokalemia can cause which symptom to occur? a. Excessive thirst b. Increased release of insulin c. Production of concentrated urine d. Decreased sensitivity to digitalis

a If prolonged, hypokalemia can lead to an inability of the kidneys to concentrate urine, causing dilute urine and excessive thirst. Potassium depletion depresses the release of insulin and results in glucose intolerance. Decreased sensitivity to digitalis does not occur with hypokalemia.

A client diagnosed with hypernatremia needs fluid volume replacement. What intravenous solution would be the safest for the nurse to administer? a. 0.45% sodium chloride b. 0.9% sodium chloride c. 5% dextrose in water d. 5% dextrose in normal saline solution

a A hypotonic solution (half-strength saline) is the solution of choice and considered safer than 5% dextrose in water because it allows a gradual reduction in the serum sodium level, thereby decreasing the risk of cerebral edema. An isotonic solution (0.9%) is not desirable as a supplement because it provides Na and CL.

The nurse is caring for a geriatric client in the home setting. Due to geriatric changes decreasing thirst, the nurse is likely to see a decrease in which fluid location which contains the most body water? a. Intracellular fluid b. Extracellular fluid c. Interstitial fluid d. Intravascular fluid

a About 60% of the adult human body is water. Most body water is located within the cell (intracellular fluid). Due to several physiological changes of aging, geriatric clients have less bodily fluids.

A client is being treated in the ICU 24 hours after having a radical neck dissection completed. The client's serum calcium concentration is 7.6 mg/dL (1.9 mmol/L). Which physical examination finding is consistent with this electrolyte imbalance? a. Presence of Trousseau sign b. Slurred speech c. Negative Chvostek sign d. Muscle weakness

a After radical neck resection, a client is prone to developing hypocalcemia. Hypocalcemia is defined as a serum value <8.6 mg/dL (<2.15 mmol/L). Signs and symptoms of hypocalcemia include Chvostek sign, which consists of muscle twitching enervated by the facial nerve when the region that is about 2 cm anterior to the earlobe, just below the zygomatic arch, is tapped; and a positive Trousseau sign can be elicited by inflating a blood pressure cuff on the upper arm to about 20 mm Hg above systolic pressure; within 2 to 5 minutes, carpal spasm (an adducted thumb, flexed wrist and metacarpophalangeal joints, and extended interphalangeal joints with fingers together) will occur as ischemia of the ulnar nerve develops. Slurred speech and muscle weakness are signs of hypercalcemia.

An older adult client asks why he has two normal bowel movements within 30 to 45 minutes in the morning. Which is the nurse's best response? a. "Incomplete emptying of the bowel is expected with age." b. "Try temporarily limiting your fluid intake and see if it helps." c. "When was your last colonoscopy?" d. "If possible, try to delay defecation as long as possible."

a Age-related incomplete emptying of the bowel is expected. This is not a pathological finding, so the client's history of colonoscopies is not relevant. Older adults should not be encouraged to delay defecation or reduce fluid intake, as these actions may cause constipation.

Which is considered an isotonic solution? a. 0.9% normal saline b. Dextran in normal saline c. 0.45% normal saline d. 3% NaCl

a An isotonic solution is 0.9% normal saline (NaCl). Dextran in normal saline is a colloid solution, 0.45% normal saline is a hypotonic solution, and 3% NaCl is a hypertonic solution.

An older client skips several doses of an antihypertensive because it causes the client to cough. Which medication should the nurse identify as causing this client's symptom? a. enalapril b. terazosin c. verapamil d. felodipine

a Angiotensin-converting enzyme (ACE) inhibitors are well-tolerated antihypertensive drugs, however, they cause a cough. Enalapril is an example of an ACE inhibitor. Terazosin is an alpha-blocker and does not cause a cough. Verapamil and felodipine are calcium channel blockers. These medications do not cause a cough.

An older adult is prescribed an antibiotic to be taken three times a day. What is the best schedule for administration of the antibiotic? a. 0600, 1400, 2200 b. 0900, 1700, 0100 c. 0800, 1600, 2400 d. 1000, 1800, 0200

a Antibiotics should be administered on a regular schedule to maintain a constant blood level. Older adults tend to go to bed early and wake up earlier; therefore, avoiding waking the client up during the night is preferable.

A patient with diabetic ketoacidosis (DKA) has had a large volume of fluid infused for rehydration. What potential complication from rehydration should the nurse monitor for? a. Hypokalemia b. Hyperkalemia c. Hyperglycemia d. Hyponatremia

a Because a patient's serum potassium level may drop quickly as a result of rehydration and insulin treatment, potassium replacement must begin once potassium levels drop to normal in the patient with DKA.

Which electrolyte is a major anion in body fluid? a. Chloride b. Potassium c. Sodium d. Calcium

a Chloride is a major anion found in extracellular fluid. Potassium, sodium, and calcium are cations.

An older client is prescribed an anticonvulsant medication. What should the nurse include when teaching the client about this medication? a. avoid grapefruit juice b. expect dizziness to occur c. stop taking it if side effects occur d. increase exposure to direct sunlight

a Clients taking anticonvulsants should be advised to avoid grapefruit and grapefruit juice because grapefruit increases the risk of toxicity. Dizziness is an adverse effect of the medication and should be reported to the healthcare professional. Anticonvulsants should not be abruptly stopped. Anticonvulsants can cause photosensitivity. Exposure to direct sunlight should be avoided.

An older male client with a history of chronic heart failure is admitted to the hospital with dehydration. Which nursing measure is most important for the nurse to add to this client's plan of care? a. Strict intake and output monitoring b. Daily vital signs c. Weekly weights d. Increase fluid intake to 4L/day

a Dehydration is a serious problem in older adults. Strict intake and output measurements should be monitored by the nurse to ensure adequate kidney function and identify any complications. Vital signs should be measured more frequently than daily. The Institue of Medicine recommends fluid intake of 3.7L/day for men older than 50 years of age. This client has heart failure, so the nurse must be careful about fluid overload when rehydrating. 4L/day is higher than the recommended amount of fluid intake. The client's weight should be monitored more closely than weekly because sudden changes must be detected.

Which condition might occur with respiratory acidosis? a. Increased intracranial pressure b. Decreased blood pressure c. Decreased pulse d. Mental alertness

a If respiratory acidosis is severe, intracranial pressure may increase, resulting in papilledema and dilated conjunctival blood vessels. Increased blood pressure, increased pulse, and decreased mental alertness occur with respiratory acidosis.

Which would be included in the teaching plan for a client diagnosed with diabetes mellitus? a. An elevated blood glucose concentration contributes to complications of diabetes, such as diminished vision. b. Sugar is found only in dessert foods. c. The only diet change needed in the treatment of diabetes is to stop eating sugar. d. Once insulin injections are started in the treatment of type 2 diabetes, they can never be discontinued.

a Diabetic retinopathy is the leading cause of blindness among people between 20 and 74 years of age in the United States; it occurs in both type 1 and type 2 diabetes. When blood glucose is well controlled, the potential for complications of diabetes is reduced. Several types of foods contain sugar, including cereals, sauces, salad dressings, fruits, and fruit juices. It is not feasible, nor advisable, to remove all sources of sugar from the diet. If the diabetes had been well controlled without insulin before the period of acute stress causing the need for insulin, the client may be able to resume previous methods for control of diabetes when the stress is resolved.

When the dawn phenomenon occurs, the patient has relatively normal blood glucose until approximate what time of day? a. 3 AM b. 5 AM c. 7 AM d. 9 AM

a During the dawn phenomenon, the patient has a relatively normal blood glucose level until about 3 AM, when the level begins to rise.

A client is diagnosed with syndrome of inappropriate antidiuretic hormone (SIADH). Laboratory results reveal serum sodium level 130 mEq/L and urine specific gravity 1.030. Which nursing intervention helps prevent complications associated with SIADH? a. Restricting fluids to 800 ml/day b. Administering vasopressin as ordered c. Elevating the head of the client's bed to 90 degrees d. Restricting sodium intake to 1 gm/day

a Excessive release of antidiuretic hormone (ADH) disturbs fluid and electrolyte balance in SIADH. The excessive ADH causes an inability to excrete dilute urine, retention of free water, expansion of extracellular fluid volume, and hyponatremia. Symptomatic treatment begins with restricting fluids to 800 ml/day. Vasopressin is administered to clients with diabetes insipidus a condition in which circulating ADH is deficient. Elevating the head of the bed decreases vascular return and decreases atrial-filling pressure, which increases ADH secretion, thus worsening the client's condition. The client's sodium is low and, therefore, shouldn't be restricted.

Which of the following is a factor affecting an increase in urine osmolality? a. Syndrome of inappropriate antidiuretic hormone release (SIADH) b. Alkalosis c. Fluid volume excess d. Myocardial infarction

a Factors increasing urine osmolality include SIADH, fluid volume deficit, acidosis, and congestive heart failure. Myocardial infarction typically is not a factor that increases urine osmolality.

Which factor increases blood urea nitrogen (BUN)? a. Gastrointestinal bleeding b. Overhydration c. Decreased protein intake d. Hypothermia

a Factors that increase BUN include gastrointestinal bleeding, decreased renal function, dehydration, increased protein intake, fever, and sepsis.

The nurse should assess the patient for signs of lethargy, increasing intracranial pressure, and seizures when the serum sodium reaches what level? a. 115 mEq/L b. 130 mEq/L c. 145 mEq/L d. 160 mEq/L

a Features of hyponatremia associated with sodium loss and water gain include anorexia, muscle cramps, and a feeling of exhaustion. The severity of symptoms increases with the degree of hyponatremia and the speed with which it develops. When the serum sodium level decreases to less than 115 mEq/L (115 mmol/L), signs of increasing intracranial pressure, such as lethargy, confusion, muscle twitching, focal weakness, hemiparesis, papilledema, seizures, and death, may occur.

Which of the following would be considered a "free" item from the exchange list? a. Diet soda b. Green salad c. Medium apple d. 1 tsp olive oil

a Free items include unsweetened iced tea, diet soda, and ice water with lemon. A green salad is exchanged for 1 vegetable. A medium apple is 1 fruit; 1 tsp of olive oil is 1 fat.

An older adult reports taking aspirin first thing in the morning, before breakfast. What priority sign or symptom should the nurse assess for in this client? a. Blood in the stool b. Jaundice c. Decreased urinary output d. Joint pain

a Gastrointestinal (GI) bleeding is one of the most serious side effects of aspirin use. To prevent GI irritation, the nurse should suggest taking the aspirin after eating. Aspirin is not known to decrease kidney function or cause jaundice. Aspirin is commonly taken for joint pain.

A client reports muscle cramps in the calves and feeling "tired a lot." The client is taking ethacrynic acid (Edecrin) for hypertension. Based on these symptoms, the client will be evaluated for which electrolyte imbalance? a. hypokalemia b. hyperkalemia c. hypocalcemia d. hypercalcemia

a Hypokalemia causes fatigue, weakness, anorexia, nausea, vomiting, cardiac dysrhythmias, leg cramps, muscle weakness, and paresthesias. Many diuretics, such as ethacrynic acid (Edecrin), also waste potassium. Symptoms of hyperkalemia include diarrhea, nausea, muscle weakness, paresthesias, and cardiac dysrhythmias. Signs of hypocalcemia include tingling in the extremities and the area around the mouth and muscle and abdominal cramps. Hypercalcemia causes deep bone pain, constipation, anorexia, nausea, vomiting, polyuria, thirst, pathologic fractures, and mental changes.

A client has chronic hyponatremia, which requires weekly laboratory monitoring to prevent the client lapsing into convulsions or a coma. What is the level of serum sodium at which a client can experience these side effects? a. 114 mEq/L b. 130 mEq/L c. 135 mEq/L d. 148 mEq/L

a Hyponatremia occurs when the serum sodium level dips below 135 mEq/L. When serum sodium levels fall below 115mEq/L, mental confusion, muscular weakness, anorexia, restlessness, elevated body temperature, tachycardia, nausea, vomiting, personality changes, convulsions, or coma can occur. A serum sodium level of 148 mEq/L would indicate hypernatremia. Normal serum concentration levels range from 135 to 145 mEq/L.

After using complementary and alternative approaches, an older adult client continues to experience pain. What should the nurse do at this time? a. provide acetaminophen b. prepare a dose of fentanyl c. ask for a prescription for meperidine d. use another nonpharmacologic approach

a If nonpharmacological means of pain control are unsuccessful, pharmacological measures should begin with the weakest type and dose of analgesic and gradually increase so that the client's response can be evaluated. Acetaminophen should be provided. Fentanyl is a powerful opioid and should not be used first. Meperidine should be avoided because of the risk of toxic effects in the older adult client. Nonpharmacological approaches have already been tried. The client's comfort needs to be addressed.

A client with diabetes mellitus has a blood glucose level of 40 mg/dL. Which rapidly absorbed carbohydrate would be most effective? a. 1/2 cup fruit juice or regular soft drink b. 4 oz of skim milk c. 1/2 tbsp honey or syrup d. three to six LifeSavers candies

a In a client with hypoglycemia, the nurse uses the rule of 15: give 15 g of rapidly absorbed carbohydrate, wait 15 minutes, recheck the blood sugar, and administer another 15 g of glucose if the blood sugar is not above 70 mg/dL. One-half cup fruit juice or regular soft drink is equivalent to the recommended 15 g of rapidly absorbed carbohydrate. Eight ounces of skim milk is equivalent to the recommended 15 g of rapidly absorbed carbohydrate. One tablespoon of honey or syrup is equivalent to the recommended 15 g of rapidly absorbed carbohydrate. Six to eight LifeSavers candies is equivalent to the recommended 15 g of rapidly absorbed carbohydrate.

A client with diabetic ketoacidosis has been brought into the ED. Which intervention is not a goal in the initial medical treatment of diabetic ketoacidosis? a. Administer glucose. b. Monitor serum electrolytes and blood glucose levels. c. Administer isotonic fluid at a high volume. d. Administer potassium replacements.

a Insulin is given intravenously. Insulin reduces the production of ketones by making glucose available for oxidation by the tissues and by restoring the liver's supply of glycogen. As insulin begins to lower the blood glucose level, the IV solution is changed to include one with glucose. Periodic monitoring of serum electrolytes and blood glucose levels is necessary. Isotonic fluid is instilled at a high volume, for example, 250 to 500 mL/hour for several hours. The rate is adjusted once the client becomes rehydrated and diuresis is less acute. Potassium replacements are given despite elevated serum levels to raise intracellular stores.

When caring for a client who has risk factors for fluid and electrolyte imbalances, which assessment finding is the highest priority for the nurse to follow up? a. Irregular heart rate b. Weight loss of 4 lb c. Mild confusion d. Blood pressure 96/53 mm Hg

a Irregular heart rate may indicate a potentially life-threatening cardiac dysrhythmia. Potassium, magnesium, and calcium imbalances may cause dysrhythmias. Weight loss is a good indicator of the amount of fluid lost, confusion may occur with dehydration and hyponatremia, and blood pressure is slightly lower than normal (though not life threatening); in each case, following up on potential cardiac dysrhythmias is a higher priority.

Which is a by-product of fat breakdown in the absence of insulin and accumulates in the blood and urine? a. Ketones b. Creatinine c. Hemoglobin d. Cholesterol

a Ketones are by-products of fat breakdown in the absence of insulin, and they accumulate in the blood and urine. Creatinine, hemoglobin, and cholesterol are not by-products of fat breakdown.

Which of the following insulins are used for basal dosage? a. Glargine (Lantus) b. NPH (Humulin N) c. Lispro (Humalog) d. Aspart (Novolog)

a Lantus is used for basal dosage. NPH is an intermediate acting insulin, usually taken after food. Humalog and Novolog are rapid-acting insulins.

The nurse is educating the client with diabetes on setting up a sick plan to manage blood glucose control during times of minor illness such as influenza. Which is the most important teaching item to include? a. Increase frequency of glucose self-monitoring. b. Decrease food intake until nausea passes. c. Do not take insulin if not eating. d. Take half the usual dose of insulin until symptoms resolve.

a Minor illnesses such as influenza can present a special challenge to a diabetic client. The body's need for insulin increases during illness. Therefore, the client should take the prescribed insulin dose, increase the frequency of glucose monitoring, and maintain adequate fluid intake to counteract the dehydrating effects of hyperglycemia. Clear liquids and juices are encouraged. Taking less than normal dose of insulin may lead to ketoacidosis.

A client is diagnosed with syndrome of inappropriate antidiuretic hormone (SIADH). The nurse informs the client that the physician will order diuretic therapy and restrict fluid and sodium intake to treat the disorder. If the client doesn't comply with the recommended treatment, which complication may arise? a. Cerebral edema b. Hypovolemic shock c. Severe hyperkalemia d. Tetany

a Noncompliance with treatment for SIADH may lead to water intoxication from fluid retention caused by excessive antidiuretic hormone. This, in turn, limits water excretion and increases the risk for cerebral edema. Hypovolemic shock results from, severe deficient fluid volume; in contrast, SIADH causes excess fluid volume. The major electrolyte disturbance in SIADH is dilutional hyponatremia, not hyperkalemia. Because SIADH doesn't alter renal function, potassium excretion remains normal; therefore, severe hyperkalemia doesn't occur. Tetany results from hypocalcemia, an electrolyte disturbance not associated with SIADH.

A client with a 30-year history of type 2 diabetes is having an annual physical and blood work. Which test result would the physician be most concerned with when monitoring the client's treatment compliance? a. glycosylated hemoglobin b. hematocrit A1c c. postprandial glucose d. All options are correct.

a Once a client with diabetes receives a treatment regimen to follow, the physician can assess the effectiveness of treatment and the client's compliance by obtaining a hemoglobin A1c test. The results of this test reflect the amount of glucose that is stored in the hemoglobin molecule during its life span of 120 days. Normally, the level of glycosylated hemoglobin is less than 7%. Amounts of 8% or greater indicate that control of the client's blood glucose level has been inadequate during the previous 2 to 3 months.

What foods can the nurse recommend for the patient with hyperkalemia? a. Apples and berries b. Green, leafy vegetables c. Milk and yogurt d. Nuts and legumes

a Sources of potassium include fruit juices and bananas, melon, citrus fruits, fresh and frozen vegetables, lean meats, milk, and whole grains

The client's lab values are sodium 166 mEq/L, potassium 5.0 mEq/L, chloride 115 mEq/L, and bicarbonate 35 mEq/L. What condition is this client likely to have, judging by anion gap? a. Metabolic acidosis b. Respiratory alkalosis c. Metabolic alkalosis d. Respiratory acidosis

a The anion gap is the difference between sodium and potassium cations and the sum of chloride and bicarbonate anions. An anion gap that exceeds 16 mEq/L indicates metabolic acidosis. In this case, the anion gap is (166 + 5) − (115 + 35), yielding 21 mEq/L, which suggests metabolic acidosis. Anion gap is not used to check for respiratory alkalosis, metabolic alkalosis, or respiratory acidosis.

The nurse is assigned to care for a client with a serum phosphorus concentration of 5.0 mg/dL (1.61 mmol/L). The nurse anticipates that the client will also experience which electrolyte imbalance? a. Hypocalcemia b. Hyperchloremia c. Hypermagnesemia d. Hyponatremia

a The client is experiencing an elevated serum phosphorus concentration. Hyperphosphatemia is defined as a serum phosphorus that exceeds 4.5 mg/dL (1.45 mmol/L). Because of the reciprocal relationship between phosphorus and calcium, a high serum phosphorus concentration tends to cause a low serum calcium concentration.

An older adult client who lives alone has been diagnosed with type 1 diabetes. To ensure safe medication use, the nurse should assess the client's: a. dexterity. b. body weight. c. bilateral muscle strength. d. vision.

a The client is likely to need to self-inject, so dexterity is imperative. Weight, strength and vision are valid components of overall assessment, but the client's dexterity relates directly to administration of injections.

Which arterial blood gas (ABG) result would the nurse anticipate for a client with a 3-day history of vomiting? a. pH: 7.55, PaCO2: 60 mm Hg, HCO3-: 28 b. pH: 7.45, PaCO2: 32 mm Hg, HCO3-: 21 c. pH: 7.28, PaCO2: 25 mm Hg, HCO3: 15 d. pH: 7.34, PaCO2: 60 mm Hg, HCO3: 34

a The client's ABG would likely demonstrate metabolic alkalosis. Metabolic alkalosis is a clinical disturbance characterized by a high pH (decreased H+ concentration) and a high plasma bicarbonate concentration. It can be produced by a gain of bicarbonate or a loss of H+. A common cause of metabolic alkalosis is vomiting or gastric suction with loss of hydrogen and chloride ions. The disorder also occurs in pyloric stenosis, where only gastric fluid is lost. The other results do not represent metabolic alkalosis.

An older adult client with a history of stroke and congestive heart failure demonstrates left-sided weakness, dysphasia and fatique. The caregiver shares that that the client often refuses to take medications as prescribed. Which assessment question should the nurse ask to best determine the possible cause of the nonadherence behaviors? a. "Have you noticed the client having any difficult swallowing?" b. "When did this reluctance to take the medication start?" c. "Does the client take frequent naps during the day?" d. "What have you done to attempt to get the client to take the medications?"

a The client's history of stroke with dysphagia should lead the nurse to the client's swallowing as it can affect he ability to take the oral medication. While the other options are appropriate assessment questions, none address the cause of the behaviors as directly as assessing for a problem swallowing.

The nurse notes that an older client takes an antidepressant medication. What should the nurse emphasize when reviewing this medication with the client? a. avoid all alcohol b. it is safe to take with antihistamines c. expect to experience urinary retention d. stop taking the medication if dizziness occurs

a The effects of antidepressants can be increased by alcohol. Alcohol should be avoided. Antidepressants can increase the effects of antihistamines. Urinary retention is an adverse effect that should be reported to the healthcare professional. Dizziness is an adverse effect. The medication should not be abruptly stopped.

A client with a magnesium concentration of 2.6 mEq/L (1.3 mmol/L) is being treated on a medical-surgical unit. Which treatment should the nurse anticipate will be used? a. Intravenous furosemide b. Fluid restriction c. Oral magnesium oxide d. Dialysis

a The nurse should anticipate the administration of furosemide for the treatment of hypermagnesemia. Administration of loop diuretics (e.g., furosemide) and sodium chloride or lactated Ringer intravenous solution enhances magnesium excretion in clients with adequate renal function. Fluid restriction is contraindicated. The client should be encouraged to increase fluids to promote the excretion magnesium through the urine. Magnesium oxide is contraindicated because it would further elevate the client's serum magnesium concentration. In acute emergencies, when the magnesium concentration is severely elevated, hemodialysis with a magnesium-free dialysate can reduce the serum magnesium to a safe concentration within hours.

Which instruction about insulin administration should a nurse give to a client? a. "Always follow the same order when drawing the different insulins into the syringe." b. "Shake the vials before withdrawing the insulin." c. "Store unopened vials of insulin in the freezer at temperatures well below freezing." d. "Discard the intermediate-acting insulin if it appears cloudy."

a The nurse should instruct the client to always follow the same order when drawing the different insulins into the syringe. Insulin should never be shaken because the resulting froth prevents withdrawal of an accurate dose and may damage the insulin protein molecules. Insulin should never be frozen because the insulin protein molecules may be damaged. The client doesn't need to discard intermediate-acting insulin if it's cloudy; this finding is normal.

Upon shift report, the nurse states the following laboratory values: pH, 7.44; PCO2, 30mmHg; and HCO3,21 mEq/L for a client with noted acid-base disturbances. Which acid-base imbalance do both nurses agree is the client's current state? a. Compensated respiratory alkalosis b. Uncompensated respiratory alkalosis c. Compensated metabolic acidosis d. Compensated metabolic alkalosis

a The question states that the client has a history of acid-base disturbance. The nurse would first note that the pH has returned to close to normal indicating compensation. The nurse then assess the PCO2 (normal: 35 to 45 mm Hg) and HCO3 (normal: 22 to 27mEq/L) levels. In a respiratory condition, the pH and the PCO2 move in opposite direction; thus, the pH rises and the PCO2 drops (alkalosis) or vice versa (acidosis). In a metabolic condition, the pH and the bicarbonate move in the same direction; if the pH is low, the bicarbonate level will be low, also. In this client, the pH is at the high end of normal, indicating compensation and alkalosis. The PCO2 is low, indicating a respiratory condition (opposite direction of the pH).

An older client has 50% of functioning nephrons. What should the nurse expect when reviewing this client's prescribed medications? a. reduced doses b. increased doses c. oral route prescribed d. intramuscular route prescribed

a The renal system is primarily responsible for the body's excretory functions, and among its activities is the excretion of drugs. Nephron units are decreased in number, and many of the remaining ones can be nonfunctional in older individuals. The glomerular filtration rate and tubular reabsorption are reduced. Drugs are not as quickly filtered from the bloodstream and are present in the body longer. The biological half-life, or the time necessary for half of the drug to be excreted, can increase as much as 40% and increase the risk of adverse drug reactions. The nurse should expect medication doses to be reduced. Increasing medication doses can increase the risk of adverse effects. The oral and intramuscular routes are not recommended to enhance absorption in the older client.

An older adult client expresses an interest in taking an over-the-counter fat-soluble vitamin supplement. What information should the nurse provide to help ensure the client's safety? a. "These vitamins are stored in the body longer, which can cause toxicity." b. "Fat-soluble vitamins generally cause gastrointestinal problems." c. "Taking supplements is more effective than eating foods high in fat-soluble vitamins." d. "Fat-soluble vitamins are difficult to get in a low-fat diet."

a Tissue structure can modify the distribution process of medications. Adipose tissue increases compared with lean body mass in older adults, especially in women; therefore, drugs stored in adipose tissue (i.e., fat-soluble drugs) will have increased tissue concentrations, decreased plasma concentrations, and a longer duration in the body. None of the other statements present accurate information about this issue.

It is important for a nurse to know how to calculate the corrected serum calcium level for a patient when hypocalcemia is seen along with low serum albumin levels. Calculate the corrected serum calcium when the serum calcium is 9 mg/dL and the serum albumin is 3 g/dL. a. 9.8 mg/dL b. 10.3 mg/dL c. 11 mg/dL d. 12 mg/dL

a To calculate corrected serum calcium, subtract the normal serum albumin level of 4 g/dL from the reported albumin level of 3 g/dL, multiply that value (1) by 0.8 (constant factor) and then add that result (0.8 mg) to the reported serum level of 9 mg/dL. Therefore, 9 + 0.8 = 9.8 mg/dL (corrected value). Note: a constant factor of 0.8 is used because, for every decrease in serum albumin of 1 g/dL below 4 g/dL, the total serum calcium level is underestimated by 0.8 mg/dL.

The nurse is caring for a client undergoing alcohol withdrawal. Which serum laboratory value should the nurse monitor most closely? a. Magnesium b. Calcium c. Phosphorus d. Potassium

a Chronic alcohol abuse is a major cause of symptomatic hypomagnesemia in the United States. The serum magnesium concentration should be measured at least every 2 or 3 days in clients undergoing alcohol withdrawal. The serum magnesium concentration may be normal at admission but may decrease as a result of metabolic changes, such as the intracellular shift of magnesium associated with intravenous glucose administration. Reference:

A patient with abnormal sodium losses is receiving a regular diet. How can the nurse supplement the patient's diet to provide 1,600 mg of sodium daily? a. One beef cube and 8 oz of tomato juice b. Four beef cubes and 8 oz of tomato juice c. One beef cube and 16 oz of tomato juice d. One beef cube and 12 oz of tomato juice

a For a patient with abnormal losses of sodium who can consume a general diet, the nurse encourages foods and fluids with high sodium content to control hyponatremia. For example, broth made with one beef cube contains approximately 900 mg of sodium; 8 oz of tomato juice contains approximately 700 mg of sodium. The nurse also needs to be familiar with the sodium content of parenteral fluids (see Table 13-5).

The nurse has been assigned to care for various clients. Which client is at the highest risk for a fluid and electrolyte imbalance? a. An 82-year-old client who receives all nutrition via tube feedings and whose medications include carvedilol and torsemide. b. A 45-year-old client who had a laparoscopic appendectomy 24 hours ago and is being advanced to a regular diet. c. A 79-year-old client admitted with a diagnosis of pneumonia. d. A 66-year-old client who had an open cholecystectomy with a T-tube placed that is draining 125 mL of bile per shift.

a The 82-year-old client has three risk factors: advanced age, tube feedings, and diuretic usage (torsemide). This client has the highest risk for fluid and electrolyte imbalances. The 45-year-old client has the risk factor of surgery, the 79-year-old client has the risk factor of advanced age, and the 66-year-old client has the risk factors of age and the bile drain, but none of these are the client at the highest risk.

The nurse is caring for a client with a serum potassium concentration of 6.0 mEq/L (6.0 mmol/L). The client is ordered to receive oral sodium polystyrene sulfonate and furosemide. What other order should the nurse anticipate giving? a. Discontinue the intravenous lactated Ringer solution. b. Increase the rate of the intravenous lactated Ringer solution. c. Change the lactated Ringer solution to 3% saline. d. Change the lactated Ringer solution to 2.5% dextrose.

a The lactated Ringer intravenous (IV) fluid is contributing to both the fluid volume excess and the hyperkalemia. In addition to the volume of IV fluids contributing to the fluid volume excess, lactated Ringer solution contains more sodium than daily requirements, and excess sodium worsens fluid volume excess. Lactated Ringer solution also contains potassium, which would worsen the hyperkalemia.

Upon shift report, the nurse states the following laboratory values: pH, 7.44; PCO2, 30mmHg; and HCO3,21 mEq/L for a client with noted acid-base disturbances. Which acid-base imbalance do both nurses agree is the client's current state? a. Compensated respiratory alkalosis b. Uncompensated respiratory alkalosis c. Compensated metabolic acidosis d. Compensated metabolic alkalosis

a The question states that the client has a history of acid-base disturbance. The nurse would first note that the pH has returned to close to normal indicating compensation. The nurse then assess the PCO2 (normal: 35 to 45 mm Hg) and HCO3 (normal: 22 to 27mEq/L) levels. In a respiratory condition, the pH and the PCO2 move in opposite direction; thus, the pH rises and the PCO2 drops (alkalosis) or vice versa (acidosis). In a metabolic condition, the pH and the bicarbonate move in the same direction; if the pH is low, the bicarbonate level will be low, also. In this client, the pH is at the high end of normal, indicating compensation and alkalosis. The PCO2 is low, indicating a respiratory condition (opposite direction of the pH).

Which of the following is an age-related change that may affect diabetes? Select all that apply. a. Decreased renal function b. Taste changes c. Decreased vision d. Increased bowel motility d. Increased proprioception

a, b, c Age-related changes include decreased renal function, taste changes, decreased vision, decreased bowel motility, and decreased proprioception.

Which of the following is an age-related change that may affect diabetes? Select all that apply. a. Decreased renal function b. Taste changes c. Decreased vision d. Increased bowel motility e. Increased proprioception

a, b, c Age-related changes include decreased renal function, taste changes, decreased vision, decreased bowel motility, and decreased proprioception.

The older adult client is prescribed a new atypical antipsychotic medication. What instructions should the nurse provde the caregiver to manage adverse effects of the medication? Select all that apply. a. "Assist the client from a sitting to standing position slowly." b. "Be sure to remove any trip or fall hazards from the house." c. "Notify the health care provider if the client appears to be sleeping during the day more than usual." d. "It's important to report any signs of confusion and disorientation." e. "Notify the health care provider if the client develops a rash or gastrointestinal problems."

a, b, c Because they were viewed as having a lower risk of adverse effects and greater tolerability, the atypical antipsychotics have largely replaced the conventional/typical agents. However, the atypical antipsychotics have been found to have their own set of side effects that are of concern in older adult care, such as postural hypotension, sedation, and falls. Atypical antipsychotics are often prescribed for clients with signs of dementia. Rashes and gastrointestinal problems are not specifically associated with this classification of medication. Reference:

Which of the following is a clinical manifestation of fluid volume excess (FVE)? Select all that apply. a. Distended neck veins b. Crackles in the lung fields c. Shortness of breath d. Decreased blood pressure e. Bradycardia

a, b, c Clinical manifestations of FVE include distended neck veins, crackles in the lung fields, shortness of breath, increased blood pressure, and tachycardia.

Which of the following is a characteristic of diabetic ketoacidosis (DKA)? Select all that apply. a. Elevated blood urea nitrogen (BUN) and creatinine b. Rapid onset c. More common in type 1 diabetes d. Absent ketones e. Normal arterial pH level

a, b, c DKA is characterized by an elevated BUN and creatinine, rapid onset, and it is more common in type 1 diabetes. Hyperglycemic hyperosmolar nonketotic syndrome (HHNS) is characterized by the absence of urine and serum ketones and a normal arterial pH level.

Which factors will cause hypoglycemia in a client with diabetes? Select all that apply. a. Client has not consumed food and continues to take insulin or oral antidiabetic medications. b. Client has not consumed sufficient calories. c. Client has been exercising more than usual. d. Client has been sleeping excessively. e. Client is experiencing effects of the aging process

a, b, c Hypoglycemia can occur when a client with diabetes is not eating at all and continues to take insulin or oral antidiabetic medications, is not eating sufficient calories to compensate for glucose-lowering medications, or is exercising more than usual. Excessive sleep and aging are not factors in the onset of hypoglycemia.

In which of the following medical conditions would administering IV normal saline solution be inappropriate? Select all that apply. a. Heart failure b. Pulmonary edema c. Renal impairment d. Burns e. Severe hemorrhage

a, b, c Normal saline is not used for heart failure, pulmonary edema, renal impairment, or sodium retention. It is used with administration of blood transfusions and to replace large sodium losses, as in burn injuries.

The physician has prescribed 0.9% sodium chloride IV for a hospitalized client in metabolic alkalosis. Which nursing actions are required to manage this client? Select all that apply. a. Compare ABG findings with previous results. b. Maintain intake and output records. c. Document presenting signs and symptoms. d. Administer IV bicarbonate. e. Suction the client's airway.

a, b, c Metabolic alkalosis results in increased plasma pH because of accumulated base bicarbonate or decreased hydrogen ion concentrations. The result is retention of sodium bicarbonate and increased base bicarbonate. Nursing management includes documenting all presenting signs and symptoms to provide accurate baseline data, monitoring laboratory values, comparing ABG findings with previous results (if any), maintaining accurate intake and output records to monitor fluid status, and implementing prescribed medical therapy.

An older adult client asks about natural means to promote bowel elimination. What should the nurse encourage the client to incorporate into a daily routine? Select all that apply. a. Increased fluid intake b. Increased fruit intake c. A variety of vegetables d. Regular physical activity e. Organic foods

a, b, c, d Fluid, fruits, vegetables, and activity are important to incorporate into a daily routine to promote bowel elimination. Foods do not need to be organic to enhance bowel function.

Which question(s) asked by an older adult client indicate an understanding of the safe and effective administration of "as needed" (PRN) medications? Select all that apply. a. "How should I determine if I need the medication?" b. "What is the maximum dosage for a 24-hour period?" c. "How often can I take the medication?" d. "What do I do if the symptoms do not subside?" e. "What is the generic name of the medication?"

a, b, c, d Safe and effective administration of PRN medication use is supported by asking about how often and how much medication can be taken in a 24-hour period, and when to take the medication as well as what to do if the symptoms do not go away. While knowing the medication's generic and brand names can be helpful, that knowledge is not often associated with safe and effective administration.

A nurse is assigned to care for a patient who is suspected of having type 2 diabetes. Select all the clinical manifestations that the nurse knows could be consistent with this diagnosis. a. Blurred or deteriorating vision b. Fatigue and irritability c. Polyuria and polydipsia d. Sudden weight loss and anorexia e. Wounds that heal slowly or respond poorly to treatment

a, b, c, e

The nurse is concerned that a client is developing metabolic syndrome. What did the nurse assess to make this clinical determination? Select all that apply. a. Central obesity b. Elevated blood pressure c. Elevated triglyceride level d. Nerve pain in the legs and feet e. Elevated fasting blood glucose level

a, b, c, e Metabolic syndrome is a group of conditions such as high triglycerides, low high-density lipoprotein, elevated fasting blood sugar, elevated blood pressure, and central obesity, occurring together that increase the risk of diabetes, stroke, and coronary artery disease. Nerve pain in the legs and feet is an adverse effect of uncontrolled diabetes.

A patient is brought into the emergency department with vomiting, drowsiness, and blowing respirations. The caregiver reports that the symptoms have been progressing throughout the day. The nurse suspects diabetic ketoacidosis (DKA). Which action should the nurse take first in the management of DKA? a. Give prescribed antiemetics. b. Begin fluid replacements. c. Administer prescribed dose of insulin. d. Administer bicarbonate to correct acidosis.

b Management of DKA is aimed at correcting dehydration, electrolyte loss, and acidosis before correcting the hyperglycemia with insulin.

A patient is diagnosed with type 1 diabetes. What clinical characteristics does the nurse expect to see in this patient? Select all that apply. a. Ketosis-prone b. Little endogenous insulin c. Obesity at diagnoses d. Younger than 30 years of age e. Older than 65 years of age

a, b, d Type I diabetes mellitus is associated with the following characteristics: onset any age, but usually young (<30 y); usually thin at diagnosis, recent weight loss; etiology includes genetic, immunologic, and environmental factors (e.g., virus); often have islet cell antibodies; often have antibodies to insulin even before insulin treatment; little or no endogenous insulin; need exogenous insulin to preserve life; and ketosis prone when insulin absent.

The nurse is caring for a patient with a diagnosis of hyponatremia. What nursing intervention is appropriate to include in the plan of care for this patient? (Select all that apply.) a. Assessing for symptoms of nausea and malaise b. Encouraging the intake of low-sodium liquids c. Monitoring neurologic status d. Restricting tap water intake e. Encouraging the use of salt substitute instead of salt

a, c, d For patients at risk, the nurse closely laboratory values (i.e., sodium) and be alert for GI manifestations such as anorexia, nausea, vomiting, and abdominal cramping. The nurse must be alert for central nervous system changes, such as lethargy, confusion, muscle twitching, and seizures. Neurologic signs are associated with very low sodium levels that have fallen rapidly because of fluid overloading. For a patient with abnormal losses of sodium who can consume a general diet, the nurse encourages foods and fluids with high sodium content to control hyponatremia. For example, broth made with one beef cube contains approximately 900 mg of sodium; 8 oz of tomato juice contains approximately 700 mg of sodium. If the primary problem is water retention, it is safer to restrict fluid intake than to administer sodium.

The nurse provides education for a client with renal disease regarding the effects of medication therapy. Which statement made by the client demonstrates the teaching was effective? a. "I have cut back on my daily intake of caffeine." b. "I limit the acetaminophen I take in 24 hours for my arthritis." c. "My liver disease means I need higher doses of prescribed medications." d. "I need more warfarin daily than I did before my liver problems occurred."

b A client with renal disease has a high risk of serious side effects when using acetaminophen, because clearance of the medication is decreased. This decrease in clearance places the client at risk for medication toxicity. The amount of acetaminophen taken daily should be limited.

The health care provider ordered an IV solution for a dehydrated patient with a head injury. Select the IV solution that the nurse knows would be contraindicated. a. 0.9% NaCl b. 5% DW c. 0.45% NS d. 3% NS

b A solution of D5W is an isotonic IV solution that is contraindicated in head injury because it may increase intracranial pressure.

A client with type 2 diabetes has recently been prescribed acarbose, and the nurse is explaining how to take this medication. The teaching is determined to be effective based on which statement by the client? a. "I will take this medication in the morning, 15 minutes before breakfast." b. "I will take this medication in the morning, with my first bite of breakfast." c. "This medication needs to be taken after the midday meal." d. "It does not matter what time of day I take this medication."

b Alpha-glucosidase inhibitors such as acarbose and miglitol, delay absorption of complex carbohydrates in the intestine and slow entry of glucose into systemic circulation. They must be taken with the first bite of food to be effective.

A nurse administers medications to a group of older adults in a residential facility. Which client is most likely to experience adverse effects? a. an 82-year-old woman with constipation b. a 77-year-old man with a creatinine of 3.6 mg/dl (274.50 µmol/l) c. a 78-year-old man with a body mass index of 35 d. an 84-year-old woman with a hemoglobin of 98 g/dl (980 g/l)

b Although age-related changes can influence skills related to taking medications, risk factors that commonly occur in older adults exert a stronger influence. A creatinine of 3.6 mg/dl (274.50 µmol/l) reflects renal failure, which will lead to an increase in serum levels of medications. Iron deficiency anemia, obesity, and constipation exhibit no impacts to the risk of adverse and altered effects.

A nurse admits an older adult client to the hospital with a diagnosis of failure to thrive. Which laboratory data should the nurse expect? a. calcium 14.2 mg/dl (3.55 mmol/l) and magnesium 3.2 mg/dl (1.32 mmol/l) b. albumin 2.8 g/dl (28 g/l) and red blood cells 4.1 cells/mcL (4.1 × 1012/l) c. platelets 124.000 cells/mcL (124 × 1012/l) and prothrombin time (PT) 10.9 sec d. white blood cells 14.3 cells/mcL (14.3 × 1012/l) and potassium 3.2 mEq/l (3.2 mmol/l)

b Anemia and low serum albumin levels are consistent with malnutrition. Elevated white blood cells, calcium, and magnesium and low platelets and prothrombin time are not characteristic of malnutrition.

A nurse is providing care for an older adult population. Which statement demonstrates an understanding of effective medication therapy safety? a. "The pharmacokinetics of medications are similar regardless of age or gender." b. "Beer's criteria is a valuable tool for managing potential medication risks for older adults." c. "Medication nonadherence is less of a problem among the older population." d. "Pharmacodynamics is affected by the older population's need for help affording medications."

b Beers criteria is a listing of drugs that carry high risks for older adults and criteria for potentially inappropriate medication use in older adults. Pharmacokinetics refers to the absorption, distribution, metabolism, and excretion of drugs and is effected by a variety of factors including age and gender. Pharmacodynamics relates to how the body is effected by the drug at the cellular level and in relation to the target organ.

An older adult client is prescribed thioridazine. What should the nurse add to this client's plan of care? a. limited oral fluid intake b. fall prevention strategies c. actions to reduce diarrhea d. vital signs assessment every 2 hours

b Clients taking antipsychotics are at high risk for falls due to the hypotensive and sedative effects. Nurses should implement fall prevention measures for these individuals. There is no need to limit oral fluid intake with this medication. This medication can cause constipation. There is no need to assess vital signs every 2 hours with this medication.

The nurse prepares a new medication for an older client. Which health problem should the nurse realize will enhance the absorption of the medication in this client? a. gout b. diabetes mellitus c. multiple sclerosis d. chronic obstructive pulmonary disease

b Conditions such as diabetes mellitus can increase the absorption of drugs. Gout, multiple sclerosis, and chronic obstructive pulmonary disease are not identified as health problems that affect medication absorption.

An obese Hispanic client, age 65, is diagnosed with type 2 diabetes. Which statement about diabetes mellitus is true? a. Nearly two-thirds of clients with diabetes mellitus are older than age 60. b. Diabetes mellitus is more common in Hispanics and Blacks than in Whites. c. Type 2 diabetes mellitus is less common than type 1 diabetes mellitus. d. Approximately one-half of the clients diagnosed with type 2 diabetes are obese.

b Diabetes mellitus is more common in Hispanics and Blacks than in Whites. Only about one-third of clients with diabetes mellitus are older than age 60 and 85% to 90% have type 2. At least 80% of clients diagnosed with type 2 diabetes mellitus are obese.

During a physical examination the nurse notes flaccid skin around the entrance to an older adult client's anus. What should the nurse suspect is occurring with this client? a. Rectal tumor b. Hemorrhoids c. Rectal fissure d. Anal infection

b Flaccid skin around the entrance to the anus are hemorrhoids. This tissue is not a rectal tumor. A rectal fissure is not visible when inspecting the anus. The flaccid tissue is not caused by an infection.

An older adult client has a history of a deep vein thrombosis. The client should be taught to avoid excessive intake of which vitamin or nutrient? a. Vitamin D b. Vitamin K c. Calcium d. Potassium

b High doses of vitamin K can result in the formation of blood clots. Excess vitamin D can result in calcium deposits in the kidneys and arteries. Calcium in excess can lead to kidney stones. Excess potassium can lead to cardiac abnormalities.

An older adult patient is in the hospital being treated for sepsis related to a urinary tract infection. The patient has started to have an altered sense of awareness, profound dehydration, and hypotension. What does the nurse suspect the patient is experiencing? a. Systemic inflammatory response syndrome b. Hyperglycemic hyperosmolar syndrome c. Multiple-organ dysfunction syndrome d. Diabetic ketoacidosis

b Hyperglycemic hyperosmolar syndrome (HHS) occurs most often in older people (50 to 70 years of age) who have no known history of diabetes or who have type 2 diabetes (Reynolds, 2012). The clinical picture of HHS is one of hypotension, profound dehydration (dry mucous membranes, poor skin turgor), tachycardia, and variable neurologic signs (e.g., alteration of consciousness, seizures, hemiparesis).

A patient's serum sodium concentration is within the normal range. What should the nurse estimate the serum osmolality to be? a. <136 mOsm/kg b. 275-300 mOsm/kg c. >408 mOsm/kg d. 350-544 mOsm/kg

b In healthy adults, normal serum osmolality is 270 to 300 mOsm/kg

The nurse is caring for a client in heart failure with signs of hypervolemia. Which vital sign is indicative of the disease process? a. Low heart rate b. Elevated blood pressure c. Rapid respiration d. Subnormal temperature

b Indicative of hypervolemia is a bounding pulse and elevated blood pressure due to the excess volume in the system. Respirations are not typically affected unless there is fluid accumulation in the lungs. Temperature is not generally affected.

A nurse is assessing a client with syndrome of inappropriate antidiuretic hormone. Which finding requires further action? a. Tetanic contractions b. Jugular vein distention c. Weight loss d. Polyuria

b Jugular vein distention requires further action because this finding signals vascular fluid overload. Tetanic contractions aren't associated with this disorder, but weight gain and fluid retention from oliguria are. Polyuria is associated with diabetes insipidus, which occurs with inadequate production of antidiuretic hormone.

A patient is admitted with a diagnosis of renal failure. The patient complains of "stomach distress" and describes ingesting several antacid tablets over the past 2 days. Blood pressure is 110/70 mm Hg, face is flushed, and the patient is experiencing generalized weakness. Which is the most likely magnesium level associated with the symptoms the patient is having? a. 11 mEq/L b. 5 mEq/L c. 2 mEq/L d. 1 mEq/L

b Magnesium excess (>2.7 mEq/L) is associated with the following symptoms: flushing, hypotension, muscle weakness, drowsiness, hypoactive reflexes, depressed respirations, and cardiac arrest. The respiratory center is depressed when serum magnesium levels exceed 10 mEq/L (5 mmol/L). This is not present in this patient, so the magnesium level is unlikely to be 11 mEq/L. Coma, atrioventricular heart block, and cardiac arrest can occur when the serum magnesium level is greatly elevated and not treated.

The nurse on a surgical unit is caring for a client recovering from recent surgery with the placement of a nasogastric tube to low continuous suction Which acid-base imbalance is most likely to occur? a. Respiratory alkalosis b. Metabolic alkalosis c. Respiratory acidosis d. Metabolic acidosis

b Metabolic alkalosis results in increased plasma pH because of an accumulated base bicarbonate or decreased hydrogen ion concentration. Factors that increase base bicarbonate include excessive oral or parenteral use of bicarbonate-containing drugs, a rapid decrease in extracellular fluid volume and loss of hydrogen and chloride ions as with gastric suctioning. Acidotic states are from excess carbonic acid and hydrogen ions in the system. Respiratory alkalosis results from a carbonic acid deficit that occurs when rapid breathing releases more CO2 than necessary.

A client has a respiratory rate of 38 breaths/min. What effect does breathing faster have on arterial pH level? a. No effect b. Increases arterial pH c. Decreases arterial pH d. Provides long-term pH regulation

b Respiratory alkalosis is always caused by hyperventilation, which is a decrease in plasma carbonic acid concentration. The pH is elevated above normal as a result of a low PaCO2.

Which sign suggests that a client with the syndrome of inappropriate antidiuretic hormone (SIADH) secretion is experiencing complications? a. Tetanic contractions b. Jugular vein distention c. Weight loss d. Polyuria

b SIADH secretion causes antidiuretic hormone overproduction, which leads to fluid retention. Severe SIADH can cause such complications as vascular fluid overload, signaled by jugular vein distention. This syndrome isn't associated with tetanic contractions. It may cause weight gain and fluid retention (secondary to oliguria).

An older client is prescribed a sodium-restricted diet. Which medication should the nurse anticipate being changed for this client? a. calcium carbonate b. sodium bicarbonate c. aluminum hydroxide d. hydrogen ion inhibitors

b Sodium bicarbonate can cause hypernatremia and should be avoided by clients prescribed a low-sodium diet. Calcium carbonate can lead to hypercalcemia. Aluminum hydroxide can cause hyperphosphatemia. Hydrogen ion inhibitors do not affect sodium or fluid balance.

After a health assessment an older client is scheduled for tests to determine the presence of diabetes. Which finding was most likely used to plan for this client's diagnostic testing? a. cataracts b. neuropathy c. leg cramps d. hypertension

b Some indications of diabetes in older adults include neuropathy, glaucoma, and orthostatic hypotension. Cataracts, leg cramps, and hypertension are not symptoms associated with undiagnosed diabetes.

An older adult client reports taking the antihypertensive as prescribed; however, the most recent blood pressure measurement is 170/90 mm Hg. What should the nurse consider as interfering with the effects of the antihypertensive medication? a. Insulin b. Antacid c. Digoxin d. Thiazide diuretic

b The effects of antihypertensives can be decreased by antacids. Antihypertensive drugs can increase the effects of insulin and thiazide diuretics. Digoxin is not identified as being affected by antihypertensive medications.

Clients diagnosed with hypervolemia should avoid sweet or dry food because it a. obstructs water elimination. b. increases the client's desire to consume fluid. c. can cause dehydration. d. can lead to weight gain.

b The management goal in hypervolemia is to reduce fluid volume. For this reason, fluid is rationed and the client is advised to take a limited amount of fluid when thirsty. Sweet or dry food can increase the client's desire to consume fluid. Sweet or dry food does not obstruct water elimination or cause dehydration. Weight regulation is not part of hypervolemia management except to the extent it is achieved on account of fluid reduction.

A client has just been diagnosed with type 1 diabetes. When teaching the client and family how diet and exercise affect insulin requirements, the nurse should include which guideline? a. "You'll need more insulin when you exercise or increase your food intake." b. "You'll need less insulin when you exercise or reduce your food intake." c. "You'll need less insulin when you increase your food intake." d. "You'll need more insulin when you exercise or decrease your food intake."

b The nurse should advise the client that exercise, reduced food intake, hypothyroidism, and certain medications decrease insulin requirements. Growth, pregnancy, greater food intake, stress, surgery, infection, illness, increased insulin antibodies, and certain medications increase insulin requirements.

The nurse reviews an older adult client's medication list and immediately assesses the client's cognitive function. Which medication caused the nurse to make this clinical determination? a. Aspirin b. Codeine c. Verapamil d. Colchicine

b Varying degrees of mental dysfunction often are early symptoms of adverse reactions to commonly prescribed medications for older adults, such as codeine. Mental dysfunction is not identified as being associated with aspirin, verapamil, or colchicine.

Which laboratory result does the nurse identify as a direct result of the client's hypovolemic status with hemoconcentration? a. Abnormal potassium level b. Elevated hematocrit level c. Low white blood count d. Low urine specific gravity

b When hemoconcentration occurs due to a hypovolemic state, a high ratio of blood components in relation to watery plasma occurs, thus causing an elevated hematocrit level. A high white blood cell count and urine specific gravity is also noted. Other causes of an abnormal potassium level may be present.

The nurse is reviewing the initial laboratory test results of a client diagnosed with DKA. Which of the following would the nurse expect to find? a. Blood glucose level of 250 mg/dL b. Blood pH of 6.9 c. Serum bicarbonate of 19 mEq/L d. PaCO2 of 40 mm Hg

b With DKA, blood glucose levels are elevated to 300 to 1000 mg/dL or more. Urine contains glucose and ketones. The blood pH ranges from 6.8 to 7.3. The serum bicarbonate level is decreased to levels from 0 to 15 mEq/L. The compensatory breathing pattern can lower the partial pressure of carbon dioxide in arterial blood (PaCO2) to levels of 10 to 30 mm Hg.

A patient has been involved in a traumatic accident and is hemorrhaging from multiple sites. The nurse expects that the compensatory mechanisms associated with hypovolemia would cause what clinical manifestations? (Select all that apply.) a. Hypertension b. Oliguria c. Tachycardia d. Bradycardia e. Tachypnea

b, c, e Hypovolemia, or fluid volume deficit, is indicated by decreased, not increased, blood pressure (hypotension), oliguria, tachycardia (not bradycardia), and tachypnea.

An older adult client reports heart palpitations and claims eating rhubarb daily. Which laboratory results should the nurse review? a. Calcium b. Magnesium c. Potassium d. Sodium

c An adverse effect of excessive rhubarb is hypokalemia, which causes disturbances in cardiac function. The nurse should asses the potassium level. Rhubarb does not normally affect sodium, magnesium or calcium levels.

A nurse reviews the results of an electrocardiogram (ECG) for a patient who is being assessed for hypokalemia. Which of the following would the nurse notice as the most significant diagnostic indicator? a. Widened QRS wave b. Flat P wave c. Elevated U wave d. Peaked T wave

c An elevated U wave is specific for hypokalemia. Flat or inverted T waves may also be present. The other tracings are consistent with hyperkalemia.

With which condition should the nurse expect that a decrease in serum osmolality will occur? a. Diabetes insipidus b. Hyperglycemia c. Kidney failure d. Uremia

c Failure of the kidneys results in multiple fluid and electrolyte abnormalities including fluid volume overload. If renal function is so severely impaired that pharmacologic agents cannot act efficiently, other modalities are considered to remove sodium and fluid from the body.

A nurse is caring for a client in acute renal failure. The nurse should expect hypertonic glucose, insulin infusions, and sodium bicarbonate to be used to treat: a. hypernatremia. b. hypokalemia c. hyperkalemia. d. hypercalcemia.

c Hyperkalemia is a common complication of acute renal failure. It's life-threatening if immediate action isn't taken to reverse it. Administering glucose and regular insulin, with sodium bicarbonate if necessary, can temporarily prevent cardiac arrest by moving potassium into the cells and temporarily reducing serum potassium levels. Hypernatremia, hypokalemia, and hypercalcemia don't usually occur with acute renal failure and aren't treated with glucose, insulin, or sodium bicarbonate.

A nurse is caring for a client in acute renal failure. The nurse should expect hypertonic glucose, insulin infusions, and sodium bicarbonate to be used to treat: a. hypernatremia. b. hypokalemia. c. hyperkalemia. d. hypercalcemia.

c Hyperkalemia is a common complication of acute renal failure. It's life-threatening if immediate action isn't taken to reverse it. Administering glucose and regular insulin, with sodium bicarbonate if necessary, can temporarily prevent cardiac arrest by moving potassium into the cells and temporarily reducing serum potassium levels. Hypernatremia, hypokalemia, and hypercalcemia don't usually occur with acute renal failure and aren't treated with glucose, insulin, or sodium bicarbonate.

A nurse is assessing a client's reflexes. Which condition does the nurse need to confirm when tapping the facial nerve of a client who has dysphagia? a. hypervolemia b. hypercalcemia c. hypomagnesemia d. hypermagnesemia

c If there is a unilateral spasm of facial muscles when the nurse taps over the facial muscle, it is known as Chvostek's sign, which is a sign of hypocalcemia and hypomagnesemia. The additional symptom of dysphagia reinforces the possibility of hypomagnesemia rather than hypocalcemia. A positive Chvostek's sign does not apply to hypercalcemia, hypervolemia, or hypermagnesemia.

A client with excess fluid volume and hyponatremia is in a comatose state. What are the nursing considerations concerning fluid replacement? a. Restrict fluids and salt for 24 hours. b. Correct the sodium deficit rapidly with salt. c. Administer small volumes of a hypertonic solution. d. Monitor the serum sodium for changes hourly.

c In clients with normal or excess fluid volume, hyponatremia is usually treated effectively by restricting fluid with clients who are not neurologically impaired. When the serum sodium concentration is overcorrected (exceeding 140 mEq/L) too rapidly or in the presence of hypoxia or anoxia, the client can develop neurological symptoms. However, if neurologic symptoms are severe (e.g., seizures, delirium, coma), or if the client has traumatic brain injury, it may be necessary to administer small volumes of a hypertonic sodium solution with the goal of alleviating cerebral edema. Incorrect use of these fluids is extremely dangerous, because 1 L of 3% sodium chloride solution contains 513 mEq (mmol/L) of sodium and 1 L of 5% sodium chloride solution contains 855 mEq (mmol/L) of sodium. The recommendation for hypertonic saline administration in clients with craniocerebral trauma is between 0.10 to 1.0 mL of 3% saline per kilogram of body weight per hour.

A nurse teaches an older adult about nutrition. Which statement shows the nurse that the older adult requires further teaching? a. "Certain 'fluid' pills can decrease the potassium level in my blood." b. "Anticholinergic medications can cause my intestines to work slower." c. "My over-the-counter betacarotene pill is appropriate for long-term use." d. "Alcohol intake will interfere with absorption of B-complex vitamins and vitamin C."

c Long-term betacarotene use can cause vitamin E deficiency. Paralytic ileus can occur with anticholinergic medication. Nutritional supplements and herbal preparations can affect nutrients. Alcohol interferes with the absorption of B-complex vitamins and vitamin C.

Health teaching for a patient with diabetes who is prescribed Humulin N, an intermediate NPH insulin, would include which of the following advice? a. "Your insulin will begin to act in 15 minutes." b. "You should expect your insulin to reach its peak effectiveness by 9:00 AM if you take it at 8:00 AM." c. "You should take your insulin after you eat breakfast and dinner." d. "Your insulin will last 8 hours, and you will need to take it three times a day."

c NPH (Humulin N) insulin is an intermediate-acting insulin that has an onset of 2 to 4 hours, a peak effectiveness of 4 to 12 hours, and a duration of 16 to 20 hours.

The nurse expects that a type 1 diabetic patient may receive what percentage of his or her usual morning dose of insulin preoperatively? a. 10% to 20% b. 25% to 40% c. 50% to 60% d. 85% to 90%

c One half to two thirds of the patient's usual morning dose of insulin (either intermediate-acting insulin alone or both short- and intermediate-acting insulins) is administered subcutaneously in the morning before surgery. The remainder is then administered after surgery.

Which condition leads to chronic respiratory acidosis in older adults? a. Decreased renal function b. Erratic meal patterns c. Thoracic skeletal change d. Overuse of sodium bicarbonate

c Poor respiratory exchange as the result of chronic lung disease, inactivity, or thoracic skeletal changes may lead to chronic respiratory acidosis. Decreased renal function in older adults can cause an inability to concentrate urine and is usually associated with fluid and electrolyte imbalance. A poor appetite, erratic meal patterns, inability to prepare nutritious meals, or financial circumstances may influence nutritional status, resulting in imbalances of electrolytes. Overuse of sodium bicarbonate may lead to metabolic alkalosis.

Which is a correct route of administration for potassium? a. Subcutaneous b. Intramuscular c. Oral d. IV (intravenous) push

c Potassium may be administered through the oral route. Potassium is never administered by IV push or intramuscularly to avoid replacing potassium too quickly. Potassium is not administered subcutaneously.

The nurse suspects that a patient with diabetes has developed proliferative retinopathy. The nurse confirms this by the presence of which of the following diagnostic signs? a. Decreased capillary permeability b. Microaneurysm formation c. Neovascularization into the vitreous humor d. The leakage of capillary wall fragments into surrounding areas

c Proliferative retinopathy, an ocular complication of diabetes, occurs because of the abnormal growth of new blood vessels on the retina that bleed into the vitreous and block light. Blood vessels in the vitreous form scar tissue that can pull and detach the retina. Neovascularization into the vitreous humor is considered a diagnostic sign.

After a pain assessment, the nurse is preparing to administer an opioid analgesic to an older adult. What is the priority assessment before administration? a. Blood pressure b. Temperature c. Respiratory rate d. Pulse

c Respiratory depression is the priority safety concern when administering opioid analgesics. Pulse, temperature and blood pressure are affected to a lesser degree.

An older adult states, "I just feel so full so fast, I can't eat any more." Which response is most appropriate? a. "All of us feel that way after a meal." b. "This happens when you have gall stones." c. "Slower emptying of your stomach may be the cause." d. "Make an appointment with your health care provider."

c Slight slowing of gastric emptying in older adults after ingestion of large meals leads to early sensations of fullness. Gallstone symptoms include pain, not fullness. An emergent visit to the health care provider is not indicated.

Which statement made by an older adult client indicates to the nurse that the client is no longer experiencing a anticholinergic adverse affect to the antacids being taken regularly? s. "Spicy food does not cause me heartburn anymore." b. "My skin rash has been gone for several weeks." c. "My bowel movements are much more regular now." d. "I am so pleased that I have not had a headache in several months."

c Specific adverse effects commonly associated with anticholinergic medications include falls, constipation, somnolence, urinary retention, dry mouth, and dry eyes. Indication of normal bowel function would indicate the absence of the anticholinergic effect on a prescribed medication. Antacids are often taken to deal with heartburn triggered by spicy foods. Neither skin rashes nor headaches are considered anticholinergic adverse reactions.

A nurse explains to a client that she will administer his first insulin dose in his abdomen. How does absorption at the abdominal site compare with absorption at other sites? a. Insulin is absorbed more slowly at abdominal injection sites than at other sites. b. Insulin is absorbed rapidly regardless of the injection site. c. Insulin is absorbed more rapidly at abdominal injection sites than at other sites. d. Insulin is absorbed unpredictably at all injection sites.

c Subcutaneous insulin is absorbed most rapidly at abdominal injection sites, more slowly at sites on the arms, and slowest at sites on the anterior thigh. Absorption after injection in the buttocks is less predictable.

A patient is prescribed Glucophage, an oral antidiabetic agent classified as a biguanide. The nurse knows that a primary action of this drug is its ability to: a. Stimulate the beta cells of the pancreas to secrete insulin. b. Decrease the body's sensitivity to insulin. c. Inhibit the production of glucose by the liver. d. Increase the absorption of carbohydrates in the intestines.

c The action of the biguanides can be found in Table 30-6 in the text.

A client with long-standing type 1 diabetes is admitted to the hospital with unstable angina pectoris. After the client's condition stabilizes, the nurse evaluates the diabetes management regimen. The nurse learns that the client sees the physician every 4 weeks, injects insulin after breakfast and dinner, and measures blood glucose before breakfast and at bedtime. Consequently, the nurse should formulate a nursing diagnosis of: a. Impaired adjustment. b. Defensive coping. c. Deficient knowledge (treatment regimen). d. Health-seeking behaviors (diabetes control).

c The client should inject insulin before, not after, breakfast and dinner — 30 minutes before breakfast for the a.m. dose and 30 minutes before dinner for the p.m. dose. Therefore, the client has a knowledge deficit regarding when to administer insulin. By taking insulin, measuring blood glucose levels, and seeing the physician regularly, the client has demonstrated the ability and willingness to modify his lifestyle as needed to manage the disease. This behavior eliminates the nursing diagnoses of Impaired adjustment and Defensive coping. Because the nurse, not the client, questioned the client's health practices related to diabetes management, the nursing diagnosis of Health-seeking behaviors isn't warranted.

A client with pancreatic cancer is experiencing severe pain. Which position should the nurse suggest to this client to increase comfort? a. prone b. supine c. lean forward d. left-sided semi-recumbent

c The client with pancreatic cancer may experience epigastric pain that radiates to the back. This pain is relieved when the client leans forward. The recumbent position, such as supine or left-sided semi-recumbent, causes the pain to be worse. The prone position could also cause the pain to be worse.

A nurse is monitoring a client being evaluated who has a potassium level of 7 mEq/L (mmol/L). Which electrocardiogram changes will the client display? a. prolonged T waves b. elevated ST segment c. peaked T waves d. shortened PR interval

c The earliest changes occur when the serum potassium level is 7 mEq/L (mmol/L). Cardiac tracings include peaked and narrow T waves, ST segment depression, and a shortened QT interval.

Which is the best nursing explanation for the symptom of polyuria in a client with diabetes mellitus? a. With diabetes, drinking more results in more urine production. b. Increased ketones in the urine promote the manufacturing of more urine. c. High sugar pulls fluid into the bloodstream, which results in more urine production. d. The body's requirement for fuel drives the production of urine.

c The hypertonicity from concentrated amounts of glucose in the blood pulls fluid into the vascular system, resulting in polyuria. The urinary frequency triggers the thirst response, which then results in polydipsia. Ketones in the urine and body requirements do not affect the production of urine.

The nurse is providing information about foot care to a client with diabetes. Which of the following would the nurse include? a. "Wash your feet in hot water every day." b. "Use a razor to remove corns or calluses." c. "Be sure to apply a moisturizer to feet daily." d. "Wear well-fitting comfortable rubber shoes."

c The nurse should advise the client to apply a moisturizer to the feet daily. The client should use warm water not hot water to bathe his feet. Razors to remove corns or calluses must be avoided to prevent injury and infection. The client should wear well-fitting comfortable shoes, avoiding shoes made of rubber, plastic or vinyl which would cause the feet to perspire.

An older adult client asks if it is "normal" to have to move the bowels twice within the same hour. What should the nurse respond to this client? a. "This means you are constipated." b. "This is caused by your medications." c. "This is a normal age-related change." d. "It happens to people who do not drink enough fluid."

c There is a tendency for older adults to not empty the bowel with one movement. Thirty to 45 minutes after the initial movement, the remainder of the bowel movement may need to occur. Having to move the bowels more than once in an hour does not indicate constipation. This is not caused by medication and does not occur in individuals who are fluid volume depleted.

The nurse is caring for a client being treated with isotonic IV fluid for hypernatremia. What complication of hypernatremia should the nurse continuously monitor for? a. Red blood cell crenation b. Red blood cell hydrolysis c. Cerebral edema d. Renal failure

c Treatment of hypernatremia consists of a gradual lowering of the serum sodium level by the infusion of a hypotonic electrolyte solution (e.g., 0.3% sodium chloride) or an isotonic nonsaline solution (e.g., dextrose 5% in water [D5W]). D5W is indicated when water needs to be replaced without sodium. Clinicians consider a hypotonic sodium solution to be safer than D5W because it allows a gradual reduction in the serum sodium level, thereby decreasing the risk of cerebral edema. It is the solution of choice in severe hyperglycemia with hypernatremia. A rapid reduction in the serum sodium level temporarily decreases the plasma osmolality below that of the fluid in the brain tissue, causing dangerous cerebral edema.

The nurse is concerned that a client with type 2 diabetes mellitus is at risk for cardiovascular problems. What information caused the nurse to have this concern? a. Heart rate 90 and regular b. Capillary refill 3 seconds c. Triglyceride level 200 mg/dL d. Blood pressure 130/80 mm Hg

c Triglyceride monitoring is important. People with diabetes are at risk for metabolic syndrome, characterized by the combination of high triglycerides, low high-density lipoprotein, and central obesity. The risk of premature death from cardiovascular disease is increased in persons with these factors. The American Diabetes Association recommends that people with diabetes maintain their triglyceride levels below 150 mg/dL. The client's heart rate, capillary refill time, and blood pressure are all within normal limits for a client with diabetes.

A client has been prescribed warfarin. Which client statement shows an understanding of the interaction between the medication and dietary needs? a. "I need to have sufficient protein in my diet." b. "I have switched to low-fat and fat-free dairy products." c. "I love kale but I know I cannot eat it often." d. "I have cut back on coffee; just 2 small cups a day."

c Vitamin K decreases the effectiveness of warfarin. Green leafy vegetables, such as kale, spinach, turnip greens, collards, Swiss chard, mustard greens, parsley, romaine, and green leaf lettuce are high in vitamin K. None of the other foods/beverages mentioned have a negative effect on warfarin effectiveness.

Which factor is the focus of nutrition intervention for clients with type 2 diabetes? a. Protein metabolism b. Blood glucose level c. Weight loss d. Carbohydrate intake

c Weight loss is the focus of nutrition intervention for clients with type 2 diabetes. A low-calorie diet may improve clinical symptoms, and even a mild to moderate weight loss, such as 10 to 20 pounds, may lower blood glucose levels and improve insulin action. Consistency in the total amount of carbohydrates consumed is considered an important factor that influences blood glucose level. Protein metabolism is not the focus of nutrition intervention for clients with type 2 diabetes.

A recent immigrant is being treated for multiple chronic health issues. It appears that the older client has not been medication adherent. Considering the client's social history, what factor should the nurse assess first? a. Ability to swallow oral medications effectively b. Visual acuity problems c. Cultural values related to medications d. Cost of medications

c While all the options present possible factors, the client's cultural values and influences should be assessed first. If they are not a factor in the behavior, the nurse should assess for the other options provided.

A client receives a daily injection of glargine insulin at 7:00 a.m. When should the nurse monitor this client for a hypoglycemic reaction? a. Between 8:00 and 10:00 a.m. b. Between 4:00 and 6:00 p.m. c. Between 7:00 and 9:00 p.m. d. This insulin has no peak action and does not cause a hypoglycemic reaction.

d "Peakless" basal or very long-acting insulins are approved by the U.S. Food and Drug Administration for use as a basal insulin; that is, the insulin is absorbed very slowly over 24 hours and can be given once a day. It has is no peak action.

Which category of oral antidiabetic agents exerts the primary action by directly stimulating the pancreas to secrete insulin? a. Thiazolidinediones b. Biguanides c. Alpha-glucosidase inhibitors d. Sulfonylureas

d A functioning pancreas is necessary for sulfonylureas to be effective. Thiazolidinediones enhance insulin action at the receptor site without increasing insulin secretion from the beta cells of the pancreas. Biguanides facilitate the action of insulin on peripheral receptor sites. Alpha-glucosidase inhibitors delay the absorption of glucose in the intestinal system, resulting in a lower postprandial blood glucose level.

A health care provider has prescribed a calcium supplement for an older adult. Which other important nutrient should the nurse teach the client to eat to promote calcium absorption? a. Potassium and Vitamin K b. Sodium and Vitamin B12 c. Vitamin E and potassium d. Vitamin D and magnesium

d A good intake of Vitamin D and magnesium facilitates calcium absorption. The nurse should include this in the teaching.

A client with nausea, vomiting, and abdominal cramps and distention is admitted to the health care facility. Which test result is most significant? a. Blood urea nitrogen (BUN) level of 29 mg/dl b. Serum sodium level of 132 mEq/L c. Urine specific gravity of 1.025 d. Serum potassium level of 3 mEq/L

d A serum potassium level of 3 mEq/L is below normal, indicating hypokalemia. Because hypokalemia may cause cardiac arrhythmias and asystole, it's the most significant finding. In a client with a potential fluid volume imbalance, such as from vomiting, the other options are expected but none are as life-threatening as hypokalemia. A BUN level of 29 mg/dl indicates slight dehydration. A serum sodium level of 132 mEq/L is slightly below normal but not life-threatening. A urine specific gravity of 1.025 is normal.

An older adult client with osteoarthritis takes 2 tablets of 650-mg acetaminophen 3 times per day. What should the nurse respond to this client? a. "Change the number of tablets to 1 for each dose." b. "Increase the frequency to 2 tablets every 4 hours." c. "If you have more pain, another dose can be taken." d. "This is the maximum amount you can take each day."

d Acetaminophen is a popular analgesic among older adults with mild to moderate pain. It often is recommended for the initial treatment of osteoarthritis. The total daily dose should not exceed 4,000 mg as high doses taken long term can cause irreversible hepatic necrosis. Since the client is taking 3 doses of 1300 mg each or 3900 mg/day, the nurse should explain that this is maximum amount that the client can take each day. Reducing the number of tablets may not control the client's pain. Increasing the frequency to every 4 hours and taking another dose could cause the client to experience liver necrosis and should not be done.

An older client has recently been diagnosed with a gastric ulcer and asks the nurse what over-the-counter medication he can take for a headache. What is the nurse's best suggestion? a. NSAID b. Aspirin c. Ibuprofen d. Acetaminophen

d Acetaminophen is the safest over-the-counter medication for the client to take that will not aggravate his gastric ulcer. The client may not understand what an NSAID is; in addition, aspirin and NSAIDs such as ibuprofen are contraindicated with a gastric or duodenal ulcer.

A client with chronic renal failure has a serum potassium level of 6.8 mEq/L. What should the nurse assess first? a. Blood pressure b. Respirations c. Temperature d. Pulse

d An elevated serum potassium level may lead to a life-threatening cardiac arrhythmia, which the nurse can detect immediately by palpating the pulse. In addition to assessing the client's pulse, the nurse should place the client on a cardiac monitor because an arrythmia can occur suddenly. The client's blood pressure may change, but only as a result of the arrhythmia. Therefore, the nurse should assess blood pressure later. The nurse also may delay assessing respirations and temperature because these aren't affected by the serum potassium level.

An older adult client, who has been taking the same medication for months, reports developing diarrhea and a headache over the past several days. What should the nurse consider when assessing this client further? a. Onset of a new disease b. Symptom of a former illness c. Exacerbation of a current illness d. Development of a new adverse effect

d Be aware that an older adult can develop adverse effects to drugs that have been taken for years without problems. The symptoms should be reviewed with the health care provider. The symptoms should be compared with any other symptoms of a new disease. The health care provider will be able to determine if the symptoms are those of a former illness or an exacerbation of a current illness.

The nurse learns that an older client has been taking meprobamate for several months to help reduce anxiety after the death of a spouse. What care should the nurse anticipate for this client? a. Stop the medication immediately b. Give the medication with a diuretic c. Continue providing the medication as prescribed d. Gradual reduction in doses before discontinuing the medication

d Clients who have used meprobamate for a long period of time can become physically and psychologically dependent on the drug and need to be weaned from it slowly. The medication should not be abruptly discontinued. This medication does not need to be given with a diuretic. Continuing the medication as prescribed is not recommended since this medication is not recommended for use in an older client.

The nurse notes that an older client has difficulty swallowing a bolus of food when eating. Which suggestion should the nurse make to the health care provider based upon this observation? a. Fluid restriction b. Prescribe a liquid diet c. Nasogastric tube for enteral feedings d. Referral for a speech-language pathologist

d For the client with a problem swallowing, a referral to a speech-language pathologist is essential to developing an effective plan of care. A fluid restriction would not enhance this client's ability to swallow. A liquid diet could lead to aspiration. A nasogastric tube for enteral feedings is an extreme intervention at this time.

A nurse knows to assess a patient with type 1 diabetes for postprandial hyperglycemia. The nurse knows that glycosuria is present when the serum glucose level exceeds: a. 120 mg/dL b. 140 mg/dL c. 160 mg/dL d. 180 mg/dL

d Glycosuria occurs when the renal threshold for sugar exceeds 180 mg/dL. Glycosuria leads to an excessive loss of water and electrolytes (osmotic diuresis).

An older adult states experiencing heart palpitations ever since adding a vitamin and nutrient supplement to the diet. The nurse checks the label of the client's supplement. For which vitamin or nutrient is the nurse assessing? a. Vitamin D b. Folic acid c. Calcium d. Potassium

d High doses of potassium can cause cardiac problems or even lead to cardiac arrest. Excess in vitamin D can lead to calcium deposits in the kidneys and arteries. Excess calcium can lead to kidney stones or impaired ability to absorb other minerals. Excess folic acid can mask vitamin B12 deficiency.

A client presents with muscle weakness, tremors, slow muscle movements, and vertigo. The following are the client's laboratory values: Sodium 134 mEq/L (134 mmol/L)Potassium 3.2 mEq/L (3.2 mmol/L)Chloride 111 mEq/L (111 mmol/L)Magnesium 1.1 mg/dL (0.45 mmol/L)Calcium 8.4 mg/dL (2.1 mmol/L) What fluid and electrolyte imbalance would the nurse relate to the client's findings? a. hyponatremia b. hypokalemia c. hypocalcemia d. hypomagnesemia

d Magnesium, the second most abundant intracellular cation, plays a role in both carbohydrate and protein metabolism. The most common cause of this imbalance is loss in the gastrointestinal tract. Hypomagnesemia is a value less than 1.3 mg/dL (0.45 mmol/L). Signs and symptoms include muscle weakness, tremors, irregular movements, tetany, vertigo, focal seizures, and positive Chvostek's and Trousseau's signs.

An older adult client wants to take ginkgo biloba, valsartan, and hydrochlorothiazide for hypertension. The client also takes an aspirin daily. Which statement best reflects the advice of the client's nurse? a. "Ginkgo biloba may cause postprandial hypotension in older adults." b. "Ginkgo biloba has the potential to interact with hydrochlorothiazide." c. "Ginkgo biloba can interact with valsartan, reducing its effectiveness." d. "Ginkgo biloba taken with aspirin can potentially cause a drug interaction."

d Medications that are likely to interact with herbs are warfarin, insulin, aspirin, digoxin, cyclosporine, and ticlopidine. Ginkgo biloba has the potential to increase blood glucose levels in type 2 diabetes. Its use is contraindicated with monoamine oxidase inhibitors. It is not known to have interactions with valsartan or hydrochlorothiazide

A client has been diagnosed with an intestinal obstruction and has a nasogastric tube set to low continuous suction. Which acid-base disturbance is this client at risk for developing? a. Respiratory acidosis b. Respiratory alkalosis c. Metabolic acidosis d. Metabolic alkalosis

d Metabolic alkalosis is a clinical disturbance characterized by a high pH and a high plasma bicarbonate concentration. The most common cause of metabolic alkalosis is vomiting or gastric suction with loss of hydrogen and chloride ions. Gastric fluid has an acid pH, and loss of this acidic fluid increases the alkalinity of body fluids.

The nurse suspects that an older adult client is having problems with eating. What assessment finding may cause the nurse to make this clinical determination? a. completes mouth care twice a day b. last dental appointment 3 months ago c. dentures fitted with a liner 6 months ago d. dentures in a glass of water in the bathroom

d Not wearing the dentures could indicate that they are ill-fitting or uncomfortable. Conducting mouth care, seeing a dentist regularly, and having the dentures fitted and adjusted would not indicate a problem with the dentures.

Oncotic pressure refers to the a. number of dissolved particles contained in a unit of fluid. b. excretion of substances such as glucose through increased urine output. c. amount of pressure needed to stop the flow of water by osmosis. d. osmotic pressure exerted by proteins.

d Oncotic pressure is a pulling pressure exerted by proteins such as albumin. Osmolality refers to the number of dissolved particles contained in a unit of fluid. Osmotic diuresis occurs when urine output increases as a result of excretion of substances such as glucose. Osmotic pressure is the amount of pressure needed to stop the flow of water by osmosis.

A client with a tentative diagnosis of hyperosmolar hyperglycemic nonketotic syndrome (HHNS) has a history of type 2 diabetes that is being controlled with an oral diabetic agent, tolazamide. Which laboratory test is the most important for confirming this disorder? a. Serum potassium level b. Serum sodium level c. Arterial blood gas (ABG) values d. Serum osmolarity

d Serum osmolarity is the most important test for confirming HHNS; it's also used to guide treatment strategies and determine evaluation criteria. A client with HHNS typically has a serum osmolarity of more than 350 mOsm/L. Serum potassium, serum sodium, and ABG values are also measured, but they aren't as important as serum osmolarity for confirming a diagnosis of HHNS. A client with HHNS typically has hypernatremia and osmotic diuresis. ABG values reveal acidosis, and the potassium level is variable.

An adult client is brought in to the clinic feeling thirsty with dry, sticky mucous membranes; decreased urine output; fever; a rough tongue; and lethargy. The nurse reconciles the client's medication list and notes that salt tablets had been prescribed. What would the nurse do next? a. Continue to monitor client with another appointment. b. Be prepared to administer a lactated Ringer's IV. c. Be prepared to administer a sodium chloride IV. d. Consider sodium restriction with discontinuation of salt tablets.

d The client's symptoms of feeling thirsty with dry, sticky mucous membranes; decreased urine output; fever; a rough tongue; and lethargy suggest hypernatremia. The client needs to be evaluated with serum blood tests soon; a later appointment will delay treatment. It is necessary to restrict sodium intake. Salt tablets and a sodium chloride IV will only worsen this condition. A Lactated Ringer's IV is a hypertonic IV and is not used with hypernatremia. A hypotonic solution IV may be a part of the treatment, but not along with the salt tablets.

The nurse assesses an older adults client's nutritional status. Which anthropometric measurement should the nurse use to help determine if the client is experiencing protein-calorie malnutrition? a. height b. weight c. triceps skinfold d. midarm circumference

d The midarm circumference is used to assess for protein-calorie malnutrition. A circumference below 60% is an indicator for this health problem. Height, weight, and triceps skinfold are not used to assess for protein-calorie malnutrition.

A client with diabetic ketoacidosis was admitted to the intensive care unit 4 hours ago and has these laboratory results: blood glucose level 450 mg/dl, serum potassium level 2.5 mEq/L, serum sodium level 140 mEq/L, and urine specific gravity 1.025. The client has two IV lines in place with normal saline solution infusing through both. Over the past 4 hours, his total urine output has been 50 ml. Which physician order should the nurse question? a. Infuse 500 ml of normal saline solution over 1 hour. b. Hold insulin infusion for 30 minutes. c. Add 40 mEq potassium chloride to an infusion of half normal saline solution and infuse at a rate of 10 mEq/hour. d. Change the second IV solution to dextrose 5% in water.

d The nurse should question the physician's order to change the second IV solution to dextrose 5% in water. The client should receive normal saline solution through the second IV site until his blood glucose level reaches 250 mg/dl. The client should receive a fluid bolus of 500 ml of normal saline solution. The client's urine output is low and his specific gravity is high, which reveals dehydration. The nurse should expect to hold the insulin infusion for 30 minutes until the potassium replacement has been initiated. Insulin administration causes potassium to enter the cells, which further lowers the serum potassium level. Further lowering the serum potassium level places the client at risk for life-threatening cardiac arrhythmias.

A healthy older adult has begun using diphenhydramine for allergic rhinitis. Within 1 week, the client begins to exhibit signs of confusion and disorientation. The client's spouse calls the primary care provider's office and speaks with the nurse. Which event should the nurse suspect first? a. The older adult has an overwhelming infection. b. The older adult is having transient ischemic attacks. c. The older adult has hyponatremia, leading to delirium. d. The older adult is experiencing an adverse drug effect.

d The older adult has been taking diphenhydramine, which can have an anticholinergic effect. Anticholinergic drugs can lead to medication-induced cognitive impairment. There is no reason to think that the client has hyponatremia. Rhinitis does not generally cause delirium in older adults. The client is more likely to be having an adverse reaction than transient ischemic attacks.

Which assessment finding is most important in determining nursing care for a client with diabetes mellitus? a. Respirations of 12 breaths/minute b. Cloudy urine c. Blood sugar 170 mg/dL d. Fruity breath

d The rising ketones and acetone in the blood can lead to acidosis and be detected as a fruity odor on the breath. Ketoacidosis needs to be treated to prevent further complications such as Kussmaul respirations (fast, labored breathing) and renal shutdown. A blood sugar of 170 mg/dL is not ideal but will not result in glycosuria and/or trigger the classic symptoms of diabetes mellitus. Cloudy urine may indicate a UTI.

An older client presents with decreased skin turgor; a brown, dry tongue; sunken cheeks; concentrated urine; and a blood urea value of 70 mg/dL (25 mmol/L). Based on these findings, the nurse should also assess for which priority finding? a. Elevated white blood cell count b. Increased bladder distensibility c. Constipation d. Sodium and potassium imbalances

d This client's presentation is consistent with a dehydration. Fluid and electrolyte imbalances are priority complications to assess for when an older client is diagnosed with dehydration. While constipation and infection are concerns, an elevated white blood cell count is not as life threatening as an electrolyte imbalance, especially potassium. Electrolyte imbalances can affect the heart and are priority concerns. Decreased bladder distensibility occurs with dehydration, not increased distensibility.

The nurse is adding the intake and output results for a client diagnosed with dehydration. The nurse notes a 24-hour intake of 1500 mL/day between oral fluids and intravenous solutions. The output total is calculated as 2800 mL/day from urine output, emesis, and Hemovac drainage. Which nursing action is best to maintain an acceptable fluid balance? a. Suggest a fluid restriction. b. Encourage oral fluids. c. Remove the Hemovac. d. Offer a prescribed antiemetic medication.

d When calculating the intake and output of a client, it is essential to understand that the normal average intake is 2500 mL in adults. Ranges are often noted at 1800 to 3000 mL. Because the client is vomiting, offering a prescribed antiemetic medication would decrease the output from emesis and increase the input as the client may be more accepting of oral fluids. The client should be encouraged more oral intake once vomiting has subsided, but if not possible, intravenous fluids should be increased to avoid dehydration A fluid restriction could cause dehydration. Removing the Hemovac will decrease documented output but may lead to an internal infection from fluid accumulation.


संबंधित स्टडी सेट्स

Chapter 3 Test Bank Questions BA 323 EXAM 1

View Set

Chapter 58: Nursing Management: Stroke, Med Surg chapter 66, Med Surg Chapter 67, Chapter 46: Cerebral Dysfunction

View Set

Complete Sentence or Sentence Fragment?

View Set

SCI EXAM 3 groundwater and karst topography

View Set

exam 3 chapter 8 managing transaction risks

View Set

MedSurg: Ch 11 Musculoskeletal Disorders

View Set

Advanced Time Value of Money Problems

View Set